Difference between revisions of "1980 AHSME Problems/Problem 23"
(→Solution) |
J314andrews (talk | contribs) m (→Solution 2 (Stewart's Theorem)) |
||
(3 intermediate revisions by the same user not shown) | |||
Line 9: | Line 9: | ||
\text{(E)}\ \text{not uniquely determined}</math> | \text{(E)}\ \text{not uniquely determined}</math> | ||
− | == Solution == | + | == Solution 1 == |
− | + | Let <math>A</math>, <math>B</math>, and <math>C</math> be the vertices of the right triangle, where <math>C</math> is the right angle. Let <math>a</math>, <math>b</math>, and <math>c</math> be the lengths of the sides opposite <math>A</math>, <math>B</math>, and <math>C</math>, respectively. Place <math>\triangle ABC</math> in the coordinate plane such that <math>C</math> is at the origin, <math>B</math> lies on the positive <math>x</math>-axis, and <math>A</math> lies on the positive <math>y</math>-axis. Then <math>A</math> has coordinates <math>(0, b)</math> and <math>B</math> has coordinates <math>(a, 0)</math>. | |
− | + | Let <math>M</math> and <math>N</math> be the trisection points of <math>\overline{AB}</math>, with <math>M</math> closer to <math>A</math> and <math>N</math> closer to <math>B</math>. Then <math>M</math> has coordinates <math>\left(\frac{1}{3}a, \frac{2}{3}b\right)</math> and <math>N</math> has coordinates <math>\left(\frac{2}{3}a, \frac{1}{3}b\right)</math>. | |
− | < | + | Let <math>m = CM</math> and <math>n = CN</math>. Then <math>m^2 = \frac{1}{9}a^2 + \frac{4}{9}b^2</math> and <math>n^2 = \frac{4}{9}a^2 + \frac{1}{9}b^2</math>. Adding these two equations yields <math>m^2 + n^2 = \frac{5}{9}(a^2+b^2) = \frac{5}{9}c^2</math>. |
− | + | Either <math>m = \sin x</math> and <math>n = \cos x</math>, or <math>m = \cos x</math> and <math>n = \sin x</math>. In both cases, <math>m^2 + n^2 = \sin^2 x + \cos^2 x = 1</math>. Therefore, <math>\frac{5}{9}c^2 = 1</math> and <math>c = \sqrt{\frac{9}{5}} = \boxed{(\textbf{C})\ \frac{3\sqrt{5}}{5}}</math>. | |
+ | |||
+ | -j314andrews | ||
+ | |||
+ | == Solution 2 (Stewart's Theorem) == | ||
+ | Let <math>A</math>, <math>B</math>, and <math>C</math> be the vertices of the right triangle, where <math>C</math> is the right angle. Let <math>a</math>, <math>b</math>, and <math>c</math> be the lengths of the sides opposite <math>A</math>, <math>B</math>, and <math>C</math>, respectively. Let <math>M</math> and <math>N</math> be the trisection points of <math>\overline{AB}</math>. Let <math>m = CM</math> and <math>n = CN</math>. | ||
+ | |||
+ | Using Stewart's Theorem on <math>\triangle ABC</math> with segment <math>CM</math> yields: | ||
+ | |||
+ | <cmath>m^2 \cdot c + \frac{2}{3}c \cdot \frac{1}{3}c \cdot c = a^2 \cdot \frac{1}{3}c + b^2 \cdot \frac{2}{3}c</cmath> | ||
+ | |||
+ | Dividing both sides by <math>c</math> and isolating <math>m^2</math> yields: | ||
+ | |||
+ | <cmath>m^2 = \frac{1}{3}a^2 + \frac{2}{3}b^2 - \frac{2}{9}c^2\ (\textrm{i})</cmath> | ||
+ | |||
+ | Using Stewart's Theorem on <math>\triangle ABC</math> with segment <math>CN</math> yields: | ||
− | <cmath> | + | <cmath>n^2 \cdot c + \frac{2}{3}c \cdot \frac{1}{3}c \cdot c =a^2 \cdot \frac{2}{3}c + b^2 \cdot \frac{1}{3}c</cmath> |
− | |||
− | |||
− | < | + | Dividing both sides by <math>c</math> and isolating <math>n^2</math> yields: |
− | <cmath> | + | <cmath>n^2 = \frac{2}{3}a^2 + \frac{1}{3}b^2 - \frac{2}{9}c^2\ (\textrm{ii})</cmath> |
− | < | + | Adding equations <math>(\textrm{i})</math> and <math>(\textrm{ii})</math> yields: |
− | <cmath> | + | <cmath>m^2 + n^2 = a^2 + b^2 - \frac{4}{9}c^2</cmath>. |
− | + | Either <math>m = \sin x</math> and <math>n = \cos x</math>, or <math>m = \cos x</math> and <math>n = \sin x</math>. In both cases, <math>m^2 + n^2 = \sin^2 x + \cos^2 x = 1</math>. Also, by the Pythagorean theorem, <math>a^2 + b^2 = c^2</math>. | |
− | <math>\ | + | Therefore, <math>1=c^2 - \frac{4}{9}c^2 = \frac{5}{9}c^2</math>, so <math>c^2 = \frac{9}{5}</math> and <math>c = \sqrt{\frac{9}{5}} = \boxed{(\textbf{C})\ \frac{3\sqrt{5}}{5}}</math>. |
== See also == | == See also == |
Latest revision as of 16:04, 16 August 2025
Problem
Line segments drawn from the vertex opposite the hypotenuse of a right triangle to the points trisecting the hypotenuse have lengths and
, where
is a real number such that
. The length of the hypotenuse is
Solution 1
Let ,
, and
be the vertices of the right triangle, where
is the right angle. Let
,
, and
be the lengths of the sides opposite
,
, and
, respectively. Place
in the coordinate plane such that
is at the origin,
lies on the positive
-axis, and
lies on the positive
-axis. Then
has coordinates
and
has coordinates
.
Let and
be the trisection points of
, with
closer to
and
closer to
. Then
has coordinates
and
has coordinates
.
Let and
. Then
and
. Adding these two equations yields
.
Either and
, or
and
. In both cases,
. Therefore,
and
.
-j314andrews
Solution 2 (Stewart's Theorem)
Let ,
, and
be the vertices of the right triangle, where
is the right angle. Let
,
, and
be the lengths of the sides opposite
,
, and
, respectively. Let
and
be the trisection points of
. Let
and
.
Using Stewart's Theorem on with segment
yields:
Dividing both sides by and isolating
yields:
Using Stewart's Theorem on with segment
yields:
Dividing both sides by and isolating
yields:
Adding equations and
yields:
.
Either and
, or
and
. In both cases,
. Also, by the Pythagorean theorem,
.
Therefore, , so
and
.
See also
1980 AHSME (Problems • Answer Key • Resources) | ||
Preceded by Problem 22 |
Followed by Problem 24 | |
1 • 2 • 3 • 4 • 5 • 6 • 7 • 8 • 9 • 10 • 11 • 12 • 13 • 14 • 15 • 16 • 17 • 18 • 19 • 20 • 21 • 22 • 23 • 24 • 25 • 26 • 27 • 28 • 29 • 30 | ||
All AHSME Problems and Solutions |
These problems are copyrighted © by the Mathematical Association of America, as part of the American Mathematics Competitions.